Diễn Đàn MathScopeDiễn Đàn MathScope
  Diễn Đàn MathScope
Ghi Danh Hỏi/Ðáp Thành Viên Social Groups Lịch Ðánh Dấu Ðã Ðọc

Go Back   Diễn Đàn MathScope > Sơ Cấp > Đại Số và Lượng Giác > Các Bài Toán Đã Được Giải

News & Announcements

Ngoài một số quy định đã được nêu trong phần Quy định của Ghi Danh , mọi người tranh thủ bỏ ra 5 phút để đọc thêm một số Quy định sau để khỏi bị treo nick ở MathScope nhé !

* Nội quy MathScope.Org

* Một số quy định chung !

* Quy định về việc viết bài trong diễn đàn MathScope

* Nếu bạn muốn gia nhập đội ngũ BQT thì vui lòng tham gia tại đây

* Những câu hỏi thường gặp

* Về việc viết bài trong Box Đại học và Sau đại học


Trả lời Gởi Ðề Tài Mới
 
Ðiều Chỉnh Xếp Bài
Old 01-09-2009, 05:20 PM   #1
Red Devils
+Thành Viên+
 
Red Devils's Avatar
 
Tham gia ngày: Aug 2009
Đến từ: Lớp 55CLC2, trường ĐHXD
Bài gởi: 205
Thanks: 28
Thanked 395 Times in 82 Posts
Topic về Bất đẳng thức Olympic

Mình không giỏi BĐT lắm nhưng cũng bạo gan lấp ra Topic này. Topic sẽ tập hợp những bất đẳng thức trong các kì thi Olympiad ở các nước. Có 1 vài quy đinh để Topic thêm đẹp:
1. Ghi rõ nguồn: bài trong kì thi nào, năm bao nhiêu
2. Khi post bài các bạn hãy để ý số bài và ghi rõ.
3. Bạn có thể:
i. Post đề bài và để mọi người giải
ii. Post đề bài + lời giải (của mình hoặc sưu tầm)
4. Các BĐT đều phải là BĐT đã xuất hiện (chính thức hoặc dự tuyển) trong 1 kì thi toán Olympiad nào đó. Ngoài ra các bất đẳng thức xuất hiện trên các tạp chí toán học cũng được chấp nhận.
5. Khi giải bạn hãy trích dẫn lại bài sẽ giải để mọi người tiện theo dõi.
6. Bạn có thể post mở rộng của BĐT nếu muốn.
Bây giờ chúng ta cùng bắt đầu:

Bài toán 1: (IMO Shortlist 2001)
Cho $n $ số thực tuỳ ý $x_{1},x_{2},\ldots,x_{n} $. Chứng minh rằng:
$\frac{x_{1}}{1+x_{1}^{2}}+\frac{x_{2}}{1+x_{1}^{2} +x_{2}^{2}}+\cdots+\frac{x_{n}}{1+x_{1}^{2}+\cdots +x_{n}^{2}}<\sqrt{n} $
Bài toán 2: (IMO Shortlist 2001)
Chứng minh rắng với mọi số thực dương $a, b, c $ ta có:
$\frac{a}{\sqrt{a^{2}+8bc}}+\frac{b}{\sqrt{b^{2}+8c a}}+\frac{c}{\sqrt{c^{2}+8ab}}\geq 1 $
Bài toán 3: (IMO LongList 1967)
Chứng minh rắng với mọi số thực dương $a, b, c $ ta có:
$\frac{1}{a}+\frac{1}{b}+\frac{1}{c}\leq\frac{a^{8} +b^{8}+c^{8}}{a^{3}b^{3}c^{3}} $
Bài toán 4: (14th Turkish Mathematical Olympiad, 2006)
Với mọi số thực dương $a, b, c $ thỏa mãn đẳng thức $a+b + c=1 $. Chứng minh rằng:
$\frac{1}{ab+2c^2+2c}+\frac{1}{bc+2a^2+2a}+\frac{1} {ca+2b^2+2b}\geq \frac{1}{ab+bc+ca} $

Tạm thời thế đã. Mọi người cùng giải nhé.
[RIGHT][I][B]Nguồn: MathScope.ORG[/B][/I][/RIGHT]
 

thay đổi nội dung bởi: Red Devils, 01-09-2009 lúc 09:39 PM
Red Devils is offline   Trả Lời Với Trích Dẫn
The Following 5 Users Say Thank You to Red Devils For This Useful Post:
hayhay (02-09-2009), hocvienak6 (01-09-2009), searcher (02-09-2009), tho 2010 (06-03-2010)
Old 01-09-2009, 06:18 PM   #2
Conan Edogawa
+Thành Viên+
 
Conan Edogawa's Avatar
 
Tham gia ngày: Sep 2008
Đến từ: Trường ĐH Kinh tế TP.HCM
Bài gởi: 397
Thanks: 136
Thanked 303 Times in 150 Posts
Trích:
Nguyên văn bởi Red Devils View Post
Bài toán 2: (IMO Shortlist 2001)
Chứng minh rắng với mọi số thực dương $a, b, c $ ta có:
$\frac{a}{\sqrt{a^{2}+8bc}}+\frac{b}{\sqrt{b^{2}+8c a}}+\frac{c}{\sqrt{c^{2}+8ab}}\geq 1 $
Chuẩn hóa $abc=1 $. Khi đó BĐT cần cm tương đương:$\frac{1}{\sqrt{1+8a}}+\frac{1}{\sqrt{1+8b}}+\frac{ 1}{\sqrt{1+8c}}\ge 1 $

$\Leftrightarrow \sum{\sqrt{(1+8a)(1+8b)}}\ge \sqrt{(1+8a)(1+8b)(1+8c)} $

$\Leftrightarrow 8(a+b+c)+2\sqrt{(1+8a)(1+8b)(1+8c)}\sum{\sqrt{1+8a }\ge 510} $

Do $abc=1\Rightarrow a+b+c\ge 3 $ và $(1+8a)(1+8b)(1+8c)\ge 9{{a}^{\frac{8}{9}}}{{b}^{\frac{8}{9}}}{{c}^{\frac {8}{9}}}=729 $

Và $\sum{\sqrt{1+8a}\ge \sum{\sqrt{9{{a}^{\frac{8}{9}}}}\ge 9{{\left( abc \right)}^{\frac{4}{27}}}}=9} $

Cộng tất cả lại ta được đpcm.
[RIGHT][I][B]Nguồn: MathScope.ORG[/B][/I][/RIGHT]
 
Conan Edogawa is offline   Trả Lời Với Trích Dẫn
The Following 3 Users Say Thank You to Conan Edogawa For This Useful Post:
hayhay (02-09-2009), stoan94 (25-09-2009), tho 2010 (06-03-2010)
Old 01-09-2009, 07:38 PM   #3
trungdeptrai
+Thành Viên+
 
trungdeptrai's Avatar
 
Tham gia ngày: Oct 2008
Đến từ: Trường THPT Chuyên ĐHSP HN
Bài gởi: 100
Thanks: 12
Thanked 53 Times in 27 Posts
Gửi tin nhắn qua Yahoo chát tới trungdeptrai
Icon10

Mở rộng Bài toán 2: (IMO Shortlist 2001)
Với $a,b,c\geq 0 $,và $k\geq 0 $,ta có:
$\frac{a}{\sqrt{{a}^{2}+kbc}}+\frac{b}{\sqrt{{b}^{2 }+kca}}+\frac{c}{\sqrt{{c}^{2}+kab}}\geq \frac{3}{\sqrt{1+k}} $.Mong mọi người thử...
[RIGHT][I][B]Nguồn: MathScope.ORG[/B][/I][/RIGHT]
 
__________________
Live for Maths - love Maths forever
Nếu được sống thêm một cuộc đời nữa, tôi sẽ lại làm Toán...
trungdeptrai is offline   Trả Lời Với Trích Dẫn
The Following 3 Users Say Thank You to trungdeptrai For This Useful Post:
hayhay (02-09-2009), tho 2010 (06-03-2010)
Old 01-09-2009, 09:06 PM   #4
caube94
+Thành Viên+
 
caube94's Avatar
 
Tham gia ngày: Nov 2008
Đến từ: Gia Lâm -Hà Nội
Bài gởi: 117
Thanks: 9
Thanked 38 Times in 26 Posts
Trích:
Nguyên văn bởi Conan Edogawa View Post
Chuẩn hóa $abc=1 $. Khi đó BĐT cần cm tương đương:$\frac{1}{\sqrt{1+8a}}+\frac{1}{\sqrt{1+8b}}+\frac{ 1}{\sqrt{1+8c}}\ge 1 $

$\Leftrightarrow \sum{\sqrt{(1+8a)(1+8b)}}\ge \sqrt{(1+8a)(1+8b)(1+8c)} $

$\Leftrightarrow 8(a+b+c)+2\sqrt{(1+8a)(1+8b)(1+8c)}\sum{\sqrt{1+8a }\ge 510} $

Do $abc=1\Rightarrow a+b+c\ge 3 $ và $(1+8a)(1+8b)(1+8c)\ge 9{{a}^{\frac{8}{9}}}{{b}^{\frac{8}{9}}}{{c}^{\frac {8}{9}}}=729 $

Và $\sum{\sqrt{1+8a}\ge \sum{\sqrt{9{{a}^{\frac{8}{9}}}}\ge 9{{\left( abc \right)}^{\frac{4}{27}}}}=9} $

Cộng tất cả lại ta được đpcm.
Bài 2: Dùng holder cho gọn
Bài 3:
$a^8+b^8+c^8 \ge a^2b^2c^2(ab+bc+ca) $
[RIGHT][I][B]Nguồn: MathScope.ORG[/B][/I][/RIGHT]
 
__________________
Ðừng khóc vì mọi việc đã qua, hãy cười vì mọi việc đang chờ phía trước.
caube94 is offline   Trả Lời Với Trích Dẫn
The Following User Says Thank You to caube94 For This Useful Post:
tho 2010 (06-03-2010)
Old 01-09-2009, 09:12 PM   #5
Red Devils
+Thành Viên+
 
Red Devils's Avatar
 
Tham gia ngày: Aug 2009
Đến từ: Lớp 55CLC2, trường ĐHXD
Bài gởi: 205
Thanks: 28
Thanked 395 Times in 82 Posts
Trích:
Nguyên văn bởi Conan Edogawa View Post
Chuẩn hóa $abc=1 $. Khi đó BĐT cần cm tương đương:$\frac{1}{\sqrt{1+8a}}+\frac{1}{\sqrt{1+8b}}+\frac{ 1}{\sqrt{1+8c}}\ge 1 $
Bạn làm rõ khúc này đi
------------------------------
Trích:
Nguyên văn bởi trungdeptrai View Post
Mở rộng Bài toán 2: (IMO Shortlist 2001)
Với $a,b,c\geq 0 $,và $k\geq 8 $,ta có:
$\frac{a}{\sqrt{{a}^{2}+kbc}}+\frac{b}{\sqrt{{b}^{2 }+kca}}+\frac{c}{\sqrt{{c}^{2}+kab}}\geq \frac{3}{\sqrt{1+k}} $
Nếu không nhầm thì phải là $k\geq 8 $
Lời giải:
Áp dụng BĐT Cauchy- Schwarz:
$\left(\sum \frac{a}{\sqrt{{a}^{2}+kbc}} \right)\left(\sum a\sqrt{a^2+kbc}\right)\geq \left(\sum a \right)^2 $
Tiếp tục áp dụng BĐT Cauchy- Schwarz:
${\left( {\sum {a\sqrt {{a^2} + kbc} } } \right)^2} = {\left( {\sum {\sqrt a \sqrt {{a^3} + kabc} } } \right)^2} \le \left( {\sum a } \right)\left( {\sum {({a^3} + kabc)} } \right) $
Do đó:
${\left( {\sum {\frac{a}{{\sqrt {{a^2} + kbc} }}} } \right)^2}\left( {\sum a } \right)\left( {\sum {({a^3} + kabc)} } \right) \ge {\left( {\sum {\frac{a}{{\sqrt {{a^2} + kbc} }}} } \right)^2}{\left( {\sum {a\sqrt {{a^2} + kbc} } } \right)^2} \ge {\left( {\sum a } \right)^4} $
$\Leftrightarrow {\left( {\sum {\frac{a}{{\sqrt {{a^2} + kbc} }}} } \right)^2} \ge \frac{{{{\left( {\sum a } \right)}^3}}}{{\left( {\sum {({a^3} + kabc)} } \right)}} $
Cần chứng minh:
$(k + 1){\left( {\sum a } \right)^3} \ge 9\left( {\sum {({a^3} + kabc)} } \right) $
$\Leftrightarrow \left( {k - 8} \right)\left( {{a^3} + {b^3} + {c^3}} \right) + 3\left( {k + 1} \right)\left( {a + b} \right)\left( {b + c} \right)\left( {c + a} \right) \ge 27kabc $
(đúng vì $k\geq 8 $)

Các bạn tích cực tham gia nhé, mình dự định cứ sau 20 bài sẽ tổng hợp thành 1 ebook nhỏ nhỏ, tất nhiên sẽ ghi tên các bạn tham gia post bài.
[RIGHT][I][B]Nguồn: MathScope.ORG[/B][/I][/RIGHT]
 

thay đổi nội dung bởi: Red Devils, 01-09-2009 lúc 09:34 PM Lý do: Tự động gộp bài
Red Devils is offline   Trả Lời Với Trích Dẫn
The Following 4 Users Say Thank You to Red Devils For This Useful Post:
Conan Edogawa (02-09-2009), hayhay (02-09-2009), tho 2010 (06-03-2010)
Old 01-09-2009, 09:56 PM   #6
Hung_DHSP
+Thành Viên Danh Dự+
 
Tham gia ngày: Sep 2008
Đến từ: K42 CSP K53 Kinh tế quốc dân
Bài gởi: 223
Thanks: 28
Thanked 86 Times in 63 Posts
Icon10 Kazakhstan Intl 2006

Bài toán 5: Cho $a,b,c,d $ là các số thực có tổng bẳng $0 $.
CMR:
$ (ab+ac+ad+bc+bd+cd)^{2}+12\geq 6(abc+abd+acd+bcd). $
[RIGHT][I][B]Nguồn: MathScope.ORG[/B][/I][/RIGHT]
 

thay đổi nội dung bởi: Hung_DHSP, 01-09-2009 lúc 10:42 PM
Hung_DHSP is offline   Trả Lời Với Trích Dẫn
The Following 2 Users Say Thank You to Hung_DHSP For This Useful Post:
tho 2010 (06-03-2010)
Old 01-09-2009, 10:06 PM   #7
caube94
+Thành Viên+
 
caube94's Avatar
 
Tham gia ngày: Nov 2008
Đến từ: Gia Lâm -Hà Nội
Bài gởi: 117
Thanks: 9
Thanked 38 Times in 26 Posts
Cho $a;b;c \ge 0;a^2+b^2+c^2+abc=4 $.CMR:
$0 \le ab+bc+ca-abc \le 2 $
$\color{red}{USAMO} $ 2000
[RIGHT][I][B]Nguồn: MathScope.ORG[/B][/I][/RIGHT]
 
__________________
Ðừng khóc vì mọi việc đã qua, hãy cười vì mọi việc đang chờ phía trước.
caube94 is offline   Trả Lời Với Trích Dẫn
The Following User Says Thank You to caube94 For This Useful Post:
tho 2010 (06-03-2010)
Old 01-09-2009, 10:10 PM   #8
Red Devils
+Thành Viên+
 
Red Devils's Avatar
 
Tham gia ngày: Aug 2009
Đến từ: Lớp 55CLC2, trường ĐHXD
Bài gởi: 205
Thanks: 28
Thanked 395 Times in 82 Posts
Trích:
Nguyên văn bởi Hung_DHSP View Post
Bài toán 5:(Kazakhstan Intl 2006)Cho $a,b,c,d $ là các số thực có tổng bẳng $0 $.
CMR:
$ (ab+ac+ad+bc+bd+cd)^{2}+12\geq 6(abc+abd+acd+bcd). $
Trích:
Nguyên văn bởi caube94 View Post
[U]Bài toán 6:Cho $a;b;c \ge 0;a^2+b^2+c^2+abc=4 $.CMR:
$0 \le ab+bc+ca-abc \le 2 $(\color{red}{USAMO}2000)
Ghi thứ tự bài và nguồn rõ ràng đi các bạn
[RIGHT][I][B]Nguồn: MathScope.ORG[/B][/I][/RIGHT]
 
Red Devils is offline   Trả Lời Với Trích Dẫn
The Following User Says Thank You to Red Devils For This Useful Post:
tho 2010 (06-03-2010)
Old 01-09-2009, 10:33 PM   #9
trungdeptrai
+Thành Viên+
 
trungdeptrai's Avatar
 
Tham gia ngày: Oct 2008
Đến từ: Trường THPT Chuyên ĐHSP HN
Bài gởi: 100
Thanks: 12
Thanked 53 Times in 27 Posts
Gửi tin nhắn qua Yahoo chát tới trungdeptrai
Icon10

bài 6:mình cũng không nhớ,nhưng ý tưởng sau hình như là của anh Cẩn.mình post cho mọi người tham khảo nha.
Trong ba số a-1,b-1,c-1 ;luôn có 2 ít nhất 2 số cùng dấu.Giả sử hai số đó là a-1,b-1.Ta có:$c\left(a-1 \right)\left(b-1 \right)\geq 0 $
$\Rightarrow abc\geq ac+bc-c $
Lại có:${a}^{2}+{b}^{2}\geq 2ab\Rightarrow {a}^{2}+{b}^{2}+{c}^{2}+abc\geq 2ab+{c}^{2}+abc $
$\Rightarrow ab\leq 2-c $
vậy$ab+bc+ca-abc\leq 2-c+bc+ca-\left(ac+bc-c \right)=2 $
Lại có:$ab+bc+ca-abc=ab\left(1-c \right)c\left(a+b \right)\geq 0 $
có dpcm...
[RIGHT][I][B]Nguồn: MathScope.ORG[/B][/I][/RIGHT]
 
__________________
Live for Maths - love Maths forever
Nếu được sống thêm một cuộc đời nữa, tôi sẽ lại làm Toán...
trungdeptrai is offline   Trả Lời Với Trích Dẫn
The Following 3 Users Say Thank You to trungdeptrai For This Useful Post:
Red Devils (03-09-2009), tho 2010 (06-03-2010)
Old 01-09-2009, 10:57 PM   #10
Hung_DHSP
+Thành Viên Danh Dự+
 
Tham gia ngày: Sep 2008
Đến từ: K42 CSP K53 Kinh tế quốc dân
Bài gởi: 223
Thanks: 28
Thanked 86 Times in 63 Posts
Trích:
Nguyên văn bởi caube94 View Post
Cho $a;b;c \ge 0;a^2+b^2+c^2+abc=4 $.CMR:
$0 \le ab+bc+ca-abc \le 2 $
$\color{red}{USAMO} $ 2000
Đây là USA MO 2001
Cũng là Bài toán 2.40 Sáng tạo bất đẳng thức.
[RIGHT][I][B]Nguồn: MathScope.ORG[/B][/I][/RIGHT]
 
Hung_DHSP is offline   Trả Lời Với Trích Dẫn
The Following User Says Thank You to Hung_DHSP For This Useful Post:
tho 2010 (06-03-2010)
Old 01-09-2009, 11:14 PM   #11
Conan Edogawa
+Thành Viên+
 
Conan Edogawa's Avatar
 
Tham gia ngày: Sep 2008
Đến từ: Trường ĐH Kinh tế TP.HCM
Bài gởi: 397
Thanks: 136
Thanked 303 Times in 150 Posts
Người VN post bài VN chứ nhỉ
Bài 7: (Olympic 30/4 năm 2006)
Cho a,b,c>0. Chứng minh:

$\frac{{{a}^{4}}}{{{a}^{4}}+\sqrt[3]{({{a}^{6}}+{{b}^{6}}){{({{a}^{3}}+{{c}^{3}})}^{2} }}}+\frac{{{b}^{4}}}{{{b}^{4}}+\sqrt[3]{({{b}^{6}}+{{c}^{6}}){{({{b}^{3}}+{{a}^{3}})}^{2} }}}+\frac{{{c}^{4}}}{{{c}^{4}}+\sqrt[3]{({{c}^{6}}+{{a}^{6}}){{({{c}^{3}}+{{b}^{3}})}^{2} }}}\le 1 $
[RIGHT][I][B]Nguồn: MathScope.ORG[/B][/I][/RIGHT]
 

thay đổi nội dung bởi: Conan Edogawa, 01-09-2009 lúc 11:15 PM Lý do: Tự động gộp bài
Conan Edogawa is offline   Trả Lời Với Trích Dẫn
The Following 2 Users Say Thank You to Conan Edogawa For This Useful Post:
tho 2010 (06-03-2010)
Old 02-09-2009, 01:11 AM   #12
trungdeptrai
+Thành Viên+
 
trungdeptrai's Avatar
 
Tham gia ngày: Oct 2008
Đến từ: Trường THPT Chuyên ĐHSP HN
Bài gởi: 100
Thanks: 12
Thanked 53 Times in 27 Posts
Gửi tin nhắn qua Yahoo chát tới trungdeptrai
Icon10

Bài 8(IMO Shortlist 2000)a,b,c dương thỏa mãn abc=1.CM:
$\left(a-1+\frac{1}{b} \right)\left(b-1+\frac{1}{c} \right)\left(c-1+\frac{1}{a} \right)\leq 1 $
......
[RIGHT][I][B]Nguồn: MathScope.ORG[/B][/I][/RIGHT]
 
__________________
Live for Maths - love Maths forever
Nếu được sống thêm một cuộc đời nữa, tôi sẽ lại làm Toán...
trungdeptrai is offline   Trả Lời Với Trích Dẫn
The Following 2 Users Say Thank You to trungdeptrai For This Useful Post:
tho 2010 (06-03-2010)
Old 02-09-2009, 09:02 AM   #13
ll931110
+Thành Viên+
 
Tham gia ngày: Sep 2008
Bài gởi: 94
Thanks: 14
Thanked 53 Times in 26 Posts
Trích:
Nguyên văn bởi Hung_DHSP View Post
Bài toán 5: Cho $a,b,c,d $ là các số thực có tổng bẳng $0 $.
CMR:
$ (ab+ac+ad+bc+bd+cd)^{2}+12\geq 6(abc+abd+acd+bcd). $
Lâu không làm BĐT nhưng cũng thử chút xem sao
Xét phương trình
$f(x) = (x - a)(x - b)(x - c)(x - d) = 0 $
$ \leftrightarrow x^4 + (\sum ab)x^2 - (\sum abc)x + abcd = 0 $ (do $\sum a = 0 $)
$\rightarrow f'(x) = 4x^3 + 2(\sum ab)x - (\sum abc) = 0 (1) $

Do $f(x) $ có 4 nghiệm, nên theo định lí Rolle phương trình $f'(x) = 0 $ có 3 nghiệm, giả sử các nghiệm đó là $p,q,r $
Ta có:
$f'(x) = 4(x - p)(x - q)(x - r) = 4x^3 - 4(\sum p)x^2 + 4(\sum pq)x - 4pqr = 0 (2) $

Đồng nhất (1) và (2) ta được
$p + q + r = 0, \sum ab = 2 \sum pq, \sum abc = 4pqr $

Và BĐT đã cho tương đương với
$(\sum pq)^2 + 3 \ge 6pqr $
$\leftrightarrow \sum (pq)^2 + 3 \ge 6pqr $
$f(p,q,r) = \leftrightarrow \sum (pq)^2 + 3 - 6pqr \ge 0 $

BĐT hiển nhiên đúng nếu tồn tại 1 số = 0. BĐT cũng đúng nếu trong 3 số p,q,r có 1 số âm. Do vậy ta chỉ cần xét TH còn lại: có đúng 1 số dương trong 3 số p,q,r và không mất tính tổng quát, giả sử $p > 0 $

Tính $f(p,q,r) - f(p,\frac{q + r}{2}, \frac{q + r}{2}) = (q - r)^2.(\frac{7p^2}{16} - \frac{qr}{4} + \frac{3p}{4}) \ge 0 $
Cho nên BĐT sẽ đúng nếu ta cm được
$f(p,\frac{q + r}{2}, \frac{q + r}{2}) \ge 0 $
$\leftrightarrow 3p^4 - 8p^3 + 16 = 0 $
$\leftrightarrow (p - 2)^2(3p^2 - 4p + 4) \ge 0 $ (đúng)

Vậy ta có dpcm. Dấu đẳng thức xảy ra khi $p = 2, q = r = -1 $./.
[RIGHT][I][B]Nguồn: MathScope.ORG[/B][/I][/RIGHT]
 
ll931110 is offline   Trả Lời Với Trích Dẫn
The Following 3 Users Say Thank You to ll931110 For This Useful Post:
Red Devils (03-09-2009), tho 2010 (06-03-2010)
Old 02-09-2009, 09:33 AM   #14
Red Devils
+Thành Viên+
 
Red Devils's Avatar
 
Tham gia ngày: Aug 2009
Đến từ: Lớp 55CLC2, trường ĐHXD
Bài gởi: 205
Thanks: 28
Thanked 395 Times in 82 Posts
Trích:
Nguyên văn bởi Conan Edogawa View Post
Bài toán 7: (Olympic 30/4 năm 2006)
Cho $a,b,c>0 $. Chứng minh:

$\frac{{{a}^{4}}}{{{a}^{4}}+\sqrt[3]{({{a}^{6}}+{{b}^{6}}){{({{a}^{3}}+{{c}^{3}})}^{2} }}}+\frac{{{b}^{4}}}{{{b}^{4}}+\sqrt[3]{({{b}^{6}}+{{c}^{6}}){{({{b}^{3}}+{{a}^{3}})}^{2} }}}+\frac{{{c}^{4}}}{{{c}^{4}}+\sqrt[3]{({{c}^{6}}+{{a}^{6}}){{({{c}^{3}}+{{b}^{3}})}^{2} }}}\le 1 $
Lời giải: Áp dụng BĐT Holder ta có:
$\sqrt[3]{(a^6+b^6)(a^3+c^3)^2}=\sqrt[3]{(a^6+b^6)(c^3+a^3)(c^3+a^3)}\geq \left(\sqrt[3]{a^6.c^3.c^3} \right)+\left(\sqrt[3]{b^6.a^3.a^3} \right)=a^2c^2+a^2b^2 $
Tương tự:$ \sqrt[3]{({{b}^{6}}+{{c}^{6}}){{({{b}^{3}}+{{a}^{3}})}^{2} }}\geq a^2b^2+b^2c^2 $
$\sqrt[3]{({{c}^{6}}+{{a}^{6}}){{({{c}^{3}}+{{b}^{3}})}^{2} }}\geq b^2c^2+c^2a^2 $
Suy ra: $\frac{{{a}^{4}}}{{{a}^{4}}+\sqrt[3]{({{a}^{6}}+{{b}^{6}}){{({{a}^{3}}+{{c}^{3}})}^{2} }}}+\frac{{{b}^{4}}}{{{b}^{4}}+\sqrt[3]{({{b}^{6}}+{{c}^{6}}){{({{b}^{3}}+{{a}^{3}})}^{2} }}}+\frac{{{c}^{4}}}{{{c}^{4}}+\sqrt[3]{({{c}^{6}}+{{a}^{6}}){{({{c}^{3}}+{{b}^{3}})}^{2} }}}\leq \sum \frac{{{a}^{4}}}{{{a}^{4}}+a^2c^2+a^2b^2}=\sum \frac{a^2}{a^2+b^2+c^2}=1 $

Trích:
Nguyên văn bởi trungdeptrai View Post
Bài toán 8: (IMO Shortlist 2000) Cho $a,b,c $ dương thỏa mãn $abc=1 $.CM:
$\left(a-1+\frac{1}{b} \right)\left(b-1+\frac{1}{c} \right)\left(c-1+\frac{1}{a} \right)\leq 1 $
Lời giải: Vì $abc=1 $ nên đặt $a=\frac{y}{z},b\frac{z}{x},c=\frac{x}{y} $. Bất đẳng thức cần chứng minh trở thành: $(-x+y+z)(x-y+z)(z+y-z)\leq xyz $ (Đây là BĐT Schur)
[RIGHT][I][B]Nguồn: MathScope.ORG[/B][/I][/RIGHT]
 
Red Devils is offline   Trả Lời Với Trích Dẫn
The Following 2 Users Say Thank You to Red Devils For This Useful Post:
tho 2010 (06-03-2010)
Old 02-09-2009, 10:08 AM   #15
trungdeptrai
+Thành Viên+
 
trungdeptrai's Avatar
 
Tham gia ngày: Oct 2008
Đến từ: Trường THPT Chuyên ĐHSP HN
Bài gởi: 100
Thanks: 12
Thanked 53 Times in 27 Posts
Gửi tin nhắn qua Yahoo chát tới trungdeptrai
Icon10

Bài 9(MOP 02)Với a,b,c dương.CM:
${\left(\frac{2a}{b+c} \right)}^{\frac{2}{3}}+{\left(\frac{2b}{c+a} \right)}^{\frac{2}{3}}+{\left(\frac{2c}{a+b} \right)}^{\frac{2}{3}}\geq 3 $
[RIGHT][I][B]Nguồn: MathScope.ORG[/B][/I][/RIGHT]
 
__________________
Live for Maths - love Maths forever
Nếu được sống thêm một cuộc đời nữa, tôi sẽ lại làm Toán...
trungdeptrai is offline   Trả Lời Với Trích Dẫn
The Following 2 Users Say Thank You to trungdeptrai For This Useful Post:
tho 2010 (06-03-2010)
Trả lời Gởi Ðề Tài Mới

Bookmarks

Tags
bất đẳng thức, inequalities

Ðiều Chỉnh
Xếp Bài

Quuyền Hạn Của Bạn
You may not post new threads
You may not post replies
You may not post attachments
You may not edit your posts

BB code is Mở
Smilies đang Mở
[IMG] đang Mở
HTML đang Tắt

Chuyển đến


Múi giờ GMT. Hiện tại là 08:22 AM.


Powered by: vBulletin Copyright ©2000-2024, Jelsoft Enterprises Ltd.
Inactive Reminders By mathscope.org
[page compression: 114.46 k/131.67 k (13.07%)]